Use non-breaking space (~) between 'Figure' and the figure number.
[course.git] / latex / problems / Serway_and_Jewett_8 / problem08.11.tex
1 \begin{problem*}{8.11}
2 The system shown in Figure~P8.11 consists of a light, inextensible
3 cord, light, frictionless pulleys, and blocks of equal mass.  Notice
4 that block $B$ is attached to one of the pulleys.  The system is
5 initially held at rest so that the blocks are at the same height above
6 the ground.  The blocks are then released.  Find the speed of block
7 $A$ at the moment the vertical separation of the blocks is $h$.
8 %  - pulley --     ceiling
9 %  |         |        |
10 %  |         - pulley -
11 %  |             |
12 %  A             B
13 \end{problem*}
14
15 \begin{solution}
16 Because the tension on both sides of a light pulley must match, block
17 $B$ has twice the upwards force (from the strings) as block $A$, so it
18 will rise as block $A$ drops.
19
20 Block $A$ will move twice as fast and far as block $B$ (draw a few
21 snapshots or build a little model to prove that to yourself if you
22 need to).  Therefore, when the blocks are $h$ appart, block $A$ will
23 be down $2h/3$ and block $B$ will be up $h/3$.  Conserving energy, we
24 have
25 \begin{align}
26   E_i &= E_f \\
27   K_{Ai} + U_{Ai} + K_{Bi} + U_{Bi} &= K_{Af} + U_{Af} + K_{Bf} + U_{Bf} \\
28   0 + 0 + 0 + 0
29     &= \frac{1}{2} m v_{Af}^2 + mg\frac{-2h}{3}
30        \frac{1}{2} m \p({\frac{v_{Af}}{2}})^2 + mg\frac{h}{3} \\
31   0 &= v_{Af}^2 \p({\frac{1}{2} + \frac{1}{8}}) - g\frac{h}{3} \\
32   v_{Af}^2 \frac{5}{8} &= \frac{gh}{3} \\
33   v_{Af} &= \pm\sqrt{\frac{8gh}{15}}
34     = \ans{\sqrt{\frac{8gh}{15}}}
35 \end{align}
36 where we dropped the $\pm$ because we only want the magnitude of the
37 velocity, not its direction.
38 \end{solution}